Question and Answers Forum

All Questions      Topic List

Algebra Questions

Previous in All Question      Next in All Question      

Previous in Algebra      Next in Algebra      

Question Number 7407 by Tawakalitu. last updated on 27/Aug/16

Commented by sou1618 last updated on 27/Aug/16

another solution  999^2 <1024^2 =2^(10×2) =2^(20)   so 999^2 <2^(20) <2^(999)

$${another}\:{solution} \\ $$$$\mathrm{999}^{\mathrm{2}} <\mathrm{1024}^{\mathrm{2}} =\mathrm{2}^{\mathrm{10}×\mathrm{2}} =\mathrm{2}^{\mathrm{20}} \\ $$$${so}\:\mathrm{999}^{\mathrm{2}} <\mathrm{2}^{\mathrm{20}} <\mathrm{2}^{\mathrm{999}} \\ $$

Commented by Tawakalitu. last updated on 27/Aug/16

Thanks so much

$${Thanks}\:{so}\:{much} \\ $$

Answered by sandy_suhendra last updated on 28/Aug/16

log 2^(999)  = 999×log 2 = 999×0.3=299.7  log 999^(2 ) = 2×log 999 = 2×2.99 = 5.98  log 2^(999) >log 999^2  so 2^(999 ) > 999^2

$${log}\:\mathrm{2}^{\mathrm{999}} \:=\:\mathrm{999}×{log}\:\mathrm{2}\:=\:\mathrm{999}×\mathrm{0}.\mathrm{3}=\mathrm{299}.\mathrm{7} \\ $$$${log}\:\mathrm{999}^{\mathrm{2}\:} =\:\mathrm{2}×{log}\:\mathrm{999}\:=\:\mathrm{2}×\mathrm{2}.\mathrm{99}\:=\:\mathrm{5}.\mathrm{98} \\ $$$${log}\:\mathrm{2}^{\mathrm{999}} >{log}\:\mathrm{999}^{\mathrm{2}} \:{so}\:\mathrm{2}^{\mathrm{999}\:} >\:\mathrm{999}^{\mathrm{2}} \\ $$

Commented by Tawakalitu. last updated on 27/Aug/16

Thanks so much.

$${Thanks}\:{so}\:{much}. \\ $$

Answered by FilupSmith last updated on 28/Aug/16

2^(999)  >^?   999^2     let 999=2^k   k=((log 999)/(log 2))  ∴ 2^(999) >2^(2×((log 999)/(log 2)))   999>2×((log 999)/(log 2))  499.5 > ((3log(3)+log(37))/(log(2)))  499.5>log_a (t)  a^(499.5) >t     ⇒    2^(499.5) >999      (a=2, t=999)  true  ∴LHS>RHS  ∴ 2^(999) >999^2

$$\mathrm{2}^{\mathrm{999}} \:\overset{?} {>}\:\:\mathrm{999}^{\mathrm{2}} \\ $$$$ \\ $$$$\mathrm{let}\:\mathrm{999}=\mathrm{2}^{{k}} \\ $$$${k}=\frac{\mathrm{log}\:\mathrm{999}}{\mathrm{log}\:\mathrm{2}} \\ $$$$\therefore\:\mathrm{2}^{\mathrm{999}} >\mathrm{2}^{\mathrm{2}×\frac{\mathrm{log}\:\mathrm{999}}{\mathrm{log}\:\mathrm{2}}} \\ $$$$\mathrm{999}>\mathrm{2}×\frac{\mathrm{log}\:\mathrm{999}}{\mathrm{log}\:\mathrm{2}} \\ $$$$\mathrm{499}.\mathrm{5}\:>\:\frac{\mathrm{3log}\left(\mathrm{3}\right)+\mathrm{log}\left(\mathrm{37}\right)}{\mathrm{log}\left(\mathrm{2}\right)} \\ $$$$\mathrm{499}.\mathrm{5}>\mathrm{log}_{{a}} \left({t}\right) \\ $$$${a}^{\mathrm{499}.\mathrm{5}} >{t}\:\:\:\:\:\Rightarrow\:\:\:\:\mathrm{2}^{\mathrm{499}.\mathrm{5}} >\mathrm{999}\:\:\:\:\:\:\left({a}=\mathrm{2},\:{t}=\mathrm{999}\right) \\ $$$$\mathrm{true} \\ $$$$\therefore\mathrm{LHS}>\mathrm{RHS} \\ $$$$\therefore\:\mathrm{2}^{\mathrm{999}} >\mathrm{999}^{\mathrm{2}} \\ $$

Commented by FilupSmith last updated on 28/Aug/16

my attempt at a proof without approximating  logs

$$\mathrm{my}\:\mathrm{attempt}\:\mathrm{at}\:\mathrm{a}\:\mathrm{proof}\:\mathrm{without}\:\mathrm{approximating} \\ $$$$\mathrm{logs} \\ $$

Terms of Service

Privacy Policy

Contact: info@tinkutara.com